Đến nội dung

mathforlife

mathforlife

Đăng ký: 20-06-2013
Offline Đăng nhập: 07-01-2015 - 21:58
***--

#461115 Danh sách đội tuyển các trường và các tỉnh đi thi quốc gia năm 2014

Gửi bởi mathforlife trong 31-10-2013 - 18:44

copy bên MS :D

 

đội tuyển tỉnh Khánh Hòa:
1. Nguyễn Huy Thành (nick ms là huythanhlqd)
2. Lê Tiến Đạt (nick MS hình như là rongden09871)
3. Lại Diệp Thanh Hải (nick MS là hình như ha.uyen2796 )
4. Châu Chí Trung
5. Nguyễn Quốc Bảo
6. Huỳnh Bá Nhẫn (nick MS là Nhẫn)

 

Đội tuyển trường ĐHSP HN (theo thứ tự điểm luôn):
1. Nguyễn Trường Sơn
2. Hoàng Đức Anh
3. Nguyễn Nhật Minh
4. Nguyễn Chí Hiêú
5. Trần Quốc Anh
6. Phạm Hoàng Hải
7. Chu Văn Trang
8. Nguyễn Thái Hà
9. Trần Hoàng Sơn
10. Phạm Minh Khang




#460065 Đề chọn đội tuyển thi Quốc Gia Khối chuyên ĐHSP 2013-2014

Gửi bởi mathforlife trong 26-10-2013 - 15:48

Từ pt dạng $x^2+y^2=k^2$ hoàn toàn có thê đăt $x=k.sin\alpha,y=k.cos\alpha$ bạn ạ




#459240 Đề chọn đội tuyển thi Quốc Gia Khối chuyên ĐHSP 2013-2014

Gửi bởi mathforlife trong 22-10-2013 - 18:46

Bài 4 cũng không cần cầu kì như vậy

Kí hiệu $t_i$ là số các thư viện khai thác tài liệu thứ $i$. Phản chứng là có $\geq 2$ tài liệu không được sử dụng, giả sử là $t_{2015},t_{2016}$.

Ta có các bất đẳng thức sau:

+) $\sum_{i=1}^{2014}t_i \geq 2013.1008 (1)$

cm: vế trái chính là tổng số các tài liệu khai thác bởi mỗi thư viện. Mặt khác mỗi thư viện phải khai thác ít nhất 1008 tài liệu nên bđt đúng.

+) $C_{2013}^{2}.504\geq \sum_{i=1}^{2014}C_{t_i}^{2} (2)$

cm: vế phải đếm số cặp thư viên khai thác cùng một tài liệu. Mặt khác mỗi cặp thư viện khai thác chung nhiều nhất 504 tài liệu nên bđt đúng.

 

Đặt $\sum_{i=1}^{2014}t_i=u$

Sử dụng bđt C-S ta được $C_{2013}^{2}.504\geq \sum_{i=1}^{2014}C_{t_i}^{2} \geq \frac{u^2}{4028}-\frac{u}{2}$

Từ đó $u \leq 2028600$ trái với $u \geq 2029104$ theo (1).

Vậy ta có đpcm




#459112 Đề chọn đội tuyển thi Quốc Gia Khối chuyên ĐHSP 2013-2014

Gửi bởi mathforlife trong 21-10-2013 - 20:34

Khong biet ban go nham hay lam nham nhung $(u+v)^2-4$ lam sao tao ra nhan tu $u+v-4$ duoc nhi?

 

Bai 3a): Cho $DF,DE$ lan luot cat $BE,CF$ o $R,S$. Ta da biet $AN$ vuong goc $RS$.

Ta cm $RS//PQ$.

$\frac{DS}{DP}=\frac{DS}{DC}.\frac{DC}{DP}=\frac{sin(90^o-B)}{sin(90^o-A+B)}.\frac{sin(90^o-C)}{sin(C+90^o-A)}=\frac{cosB.cosC}{sin(90^o-A+B).sin(90^o-A+C)}$

Tuong tu $\frac{DR}{DQ}=\frac{cosB.cosC}{sin(90^o-A+B).sin(90^o-A+C)}$

Vay theo dinh ly Talet $RS//PQ$

Vay $AN$ vuong goc $PQ$




#459104 Đề chọn đội tuyển thi Quốc Gia Khối chuyên ĐHSP 2013-2014

Gửi bởi mathforlife trong 21-10-2013 - 20:13


Câu 2.

Tìm tất cả nghiệm thực của hệ :
$$\left\{\begin{matrix} x+x^2y=y+2\\ (2x+y)^2+3y^2=12 \end{matrix}\right.$$

 

Huong lam bai nay cua minh:

Dat $y=2sin\alpha,2x+y=2\sqrt{3}cos\alpha$ thi $x=\sqrt{3}cos\alpha-sin\alpha$

Thay vao phuong trinh 1 ta duoc: $\sqrt{3}cos{\alpha}-sin{\alpha} + (\sqrt{3}cos{\alpha}-sin{\alpha})^2.2sin{\alpha}=2sin{\alpha}+2$

hay $\sqrt{3}cos\alpha-sin\alpha+4(cos\alpha)^2.sin\alpha-4\sqrt{3}.cos\alpha.(sin\alpha)^2=2$

hay $sin(3\alpha)+\sqrt{3}cos(3\alpha)=2$ (thay $(cos\alpha)^2=1-(sin\alpha)^2$ va $(sin\alpha)^2=1-(cos\alpha)^2$)

Den day de giai duoc roi :D




#458888 Đề thi chọn học sinh giỏi toán lớp 11-12 chuyên KHTN 2013-2014 (Vòng 2)

Gửi bởi mathforlife trong 20-10-2013 - 18:45

Đề thi chọn đội tuyển tham dự VMO 2014 THPT chuyên Khoa Học Tự Nhiên

 

Ngày thi thứ nhất: Thời gian làm bài 240'

 

Bài 1: Cho dãy số $(a_n)$ thỏa mãn: $a_1=3, a_2=17, a_3=99$ và 

      $a_{n+1}=\frac{a_n^{2}+a_{n-1}^{2}-1}{a_{n-2}}$.

Chứng minh rằng $a_{2014}+1$ là số chính phương.

 

Tu cong thuc truy hoi ta co $a_n^{2}+a_{n-1}^{2}-a_{n+1}a_{n-2}=1=a_{n-1}^{2}+a_{n-2}^{2}-a_{n}a_{n-3}$

Tu do ta co $\frac{a_{n+1}+a_{n-2}}{a_n+a_{n-3}}=\frac{a_n}{a_{n-2}}$

Lam tuong tu lui dan roi nhan lai ta duoc $\frac{a_{n+1}+a_{n-2}}{a_4+a_1}=\frac{a_n.a_{n-1}}{a_3.a_2}$

Hay $a_{n+1}+a_{n-2}=2a_n.a_{n-1}$. thay cong thuc truy hoi o de bai vao ta duoc $a_n^{2}+a_{n-1}^{2}+a_{n-2}^{2}-2a_n.a_{n-1}.a_{n-2}=1$ hay $x_n^{2}+x_{n-1}^{2}+x_{n-2}^{2}-x_n.x_{n-1}.x_{n-2}=4$ voi $x_n=2a_n$. Dang thuc nay goi nho cho ta cach dat quen thuoc sau.

Viet $x_n=y_n+\frac{1}{y_n}$ voi $y_n>1$ thi ta se duoc $y_{n+1}=y_n.y_{n-1}$

Chu y $y_2=y_1^{2}$.

Ta se duoc $y_n=y_1^{F_n}$ voi $F_1=1,F_2=2,F_{n+2}=F_{n+1}+F_n$

$y_1=3+2\sqrt{2}=(\sqrt{2}+1)^2=(\alpha)^2$

Do do $2(a_{2014}+1)=x_{2014}+2=(\alpha)^{2F_{2014}}+\frac{1}{(\alpha)^{2F_{2014}}}+2=((\alpha)^{F_{2014}}+\frac{1}{(\alpha)^{F_{2014}}})^2$

Tu do $a_{2014}+1$ thuoc CP.




#455623 Tìm da thức $f(x)$ bậc không nhỏ hơn 2 thỏa mãn $f(r_i)=0$ .

Gửi bởi mathforlife trong 06-10-2013 - 13:10

Gia su $degf =n> 2$

Dat $x_i=\alpha r_i + (1-\alpha)r_{i+1} \forall i=1,...,n-1$

Tu gia thiet  ta co $\sum_{j=1}^{n}\frac{1}{x_i-r_j}=0 \forall 0<i<n$

$\frac{1}{x_1-r_1}=\sum_{j=2}^{n}\frac{1}{r_j-x_1}>\frac{1}{r_2-x_1}\Rightarrow \alpha > \frac{1}{2}$

 

$\frac{1}{r_n-x_{n-1}}=\sum_{j=1}^{n-1}\frac{1}{x_{n-1}-r_j}>\frac{1}{x_{n-1}-r_{n-1}}\Rightarrow \alpha < \frac{1}{2}$, vo ly

Vay $n=2,\alpha = \frac{1}{2}$ va $f(x)$ la da thuc bac hai co hai nghiem phan biet bat ki




#449389 Tìm đa thức $P(x)$ biết rằng $P^{2}(x)\equiv[P(...

Gửi bởi mathforlife trong 11-09-2013 - 13:36

Chăc đê phải là $(P(x))^2=P(x^2)$ chư nhỉ.

có thê xem bài 12 ơ đây http://www.imomath.c...tions=627&lmm=0




#449380 Tìm $limu_{n}$ với$u_{n}=(n+1)^3x_{n...

Gửi bởi mathforlife trong 11-09-2013 - 11:57

từ công thưc truy hôì ta có

$x_n=x_{n-1}\frac{(n-1)^2}{n(n+1)}=...$

do đó $x_n=\frac{4}{n^2(n+1)}$

khi đó $lim u_n=4$




#443966 XĐ CTTQ

Gửi bởi mathforlife trong 18-08-2013 - 22:57

vấn đề là sau khi giản ước thì $x_0,x_1$ đâu thoả mãn ct :D chắc cái tử phải là $x_{n+1}.x_{n}$

câu sau thì cậu quy nạp $x_n=2^{u_n}$ với $u_0=0,u_1=1,u_n=3^{n-2}-u_{n-1}$ :D




#443668 Tìm số nguyên dương n nhỏ nhất thỏa mãn điều kiện

Gửi bởi mathforlife trong 17-08-2013 - 17:00

Nêú $n\leq834$ thì xét $A=\left \{ 166,167,...,999 \right \}$ suy ra vô lý. vâỵ $n \geq 835$

 

Ta cm $n=835$ thoả mãn

Gọi $c,d$ lân lươt là sô nhỏ nhât và lơn nhât trong $A$.

Ta có $d-c \geq 834, c \leq 165$

$m=d-3c=(d-c)-2c \geq 834 - 2.165 > 3.166$

Xét căp sô $(m-2i,i) , i=1,2,...,166, i \neq c$

dê thâý $330$ sô trong $165$ căp sô này khác nhau và khác $c$ và tôn tại nhiêù nhât $164$ trong $330$ sô không thuôc $A$.

Suy ra tôn tại $1 \leq b \leq 166, b \neq c$ sao cho $m-2b,b \in A$

$a=m-2b \rightarrow a+2b+3c=d$

vâỵ $n min = 835$




#442925 Cho dãy số xn thoả mãn: $x_{1}\epsilon R;x_{n+1...

Gửi bởi mathforlife trong 15-08-2013 - 00:08

Tìm thâý lơì giải bên mathlinks :D http://www.artofprob...p?f=36&t=548659




#442523 $u_{n}=\frac{u_{n-1}\times u_{n-...

Gửi bởi mathforlife trong 13-08-2013 - 17:10

Từ gt $u_4=2+k$ mà $u_4 \in Z$ nên $k \in Z$

từ công thức truy hồi $u_{n}u_{n-3} - u_{n-1}u_{n-2}=k=u_{n-1}u_{n-4} - u_{n-2}u_{n-3}$

hay $ \frac {u_{n} + u_{n-2}}{u_{n-1}}=\frac{u_{n-2}+u_{n-4}}{u_{n-3}}$

suy ra $\frac{u_{2k}+u_{2k-2}}{u_{2k-1}}=\frac{u_4+u_2}{u_3}=\frac{3+k}{2}$

$\frac{u_{2k+1}+u_{2k-1}}{u_{2k}}=\frac{u_3+u_1}{u_2}=3$

suy ra $u_{2k}=\frac{3+k}{2}u_{2k-1}-u_{2k-2}$ (1)

$u_{2k+1}=3u_{2k}-u_{2k-1}$ (2)

Nếu k lẻ thì hiển nhiên $u_n \in Z \forall n$

giả sử tồn tại k chẵn thoả mãn. từ (1) suy ra $u_n \equiv 0 (mod2) \forall n\equiv 1(mod2)$

kết hợp (2) suy ra $u_n \equiv 0 (mod2) \forall n\equiv 0(mod2)$

lại kết hợp (1) suy ra $u_n \equiv 0 (mod4) \forall n\equiv 1(mod2)$

kết hợp (2) suy ra $u_n \equiv 0 (mod4) \forall n\equiv 0(mod2)$

Tiếp tục lập luận như vậy ta sẽ suy ra được điều vô lý.

Vậy đáp số bài toán là $k \in Z, k \equiv 1(mod2)$

 

P/s: không biết đúng khôgn nữa :D




#440872 $f(x^2(z^2+1)+f(y)(z+1))=1-f(z)(x^2+f(y))-z((1+z)x^2+2f(y)),\forall...

Gửi bởi mathforlife trong 06-08-2013 - 21:00

Tìm tất cả các hàm $f:R\rightarrow R$ thoả mãn:

$f(x^2(z^2+1)+f(y)(z+1))=1-f(z)(x^2+f(y))-z((1+z)x^2+2f(y)),\forall x,y,z \in R$




#436283 $x_{n}-y_{n}-z_{n}=2^{n}$

Gửi bởi mathforlife trong 19-07-2013 - 20:45

Qui nạp cttq:

 

$x_n=\frac{25.2^n+147.45^n}{43}$

$y_n=\frac{-12.2^n+98.45^n}{43}$

$z_n=\frac{-6.2^n+49.45^n}{43}$

 

từ đó hiển nhiên $x_n-y_n-z_n=2^n$